Suites et séries

Exercices du dossier Suites et séries

Exercice 1329

9 novembre 2022 12:19 — Par Patrice Lassère

Bibliographie



[ID: 2531] [Date de publication: 9 novembre 2022 12:19] [Catégorie(s): Géométrie Topologie Continuité Dérivabilité Intégration Suites et séries Suites et séries de fonctions, séries entières Fonctions holomorphes Calcul différentiel Equations différentielles Analyse fonctionnelle Combinatoires et probabilités En cours... ] [ Nombre commentaires: 0] [nombre d'éditeurs: 1 ] [Editeur(s): Emmanuel Vieillard-Baron ] [nombre d'auteurs: 1 ] [Auteur(s): Patrice Lassère ]
Accordéon
Titre
Solution
Texte

Convergence de \(\sum_n a_n^{-3}\), où \(\vert a_n-a_m\vert>1,\ \forall\,m\neq n\in\mathbb N\) *

9 novembre 2022 21:54 — Par Patrice Lassère

Soit \((a_n)_n\subset\mathbb C^\star\) une suite de nombres complexes vérifiant \[\forall\,m\neq n\in\mathbb N\quad:\quad\vert a_n-a_m\vert\geq 1.{(\text{$\star$})}\] Montrer que la série \[\sum_{n\geq 0}\dfrac{1}{a_n^3}\] converge.



[ID: 2837] [Date de publication: 9 novembre 2022 21:54] [Catégorie(s): Suites et séries ] [ Nombre commentaires: 0] [nombre d'éditeurs: 1 ] [Editeur(s): Emmanuel Vieillard-Baron ] [nombre d'auteurs: 1 ] [Auteur(s): Patrice Lassère ]
Accordéon
Titre
Solution
Texte

Convergence de \(\sum_n a_n^{-3}\), où \(\vert a_n-a_m\vert>1,\ \forall\,m\neq n\in\mathbb N\)
Par Patrice Lassère le 9 novembre 2022 21:54

Considérons une suite \((a_n)_n\) de nombres complexes vérifiant la propriété (\(\star\)). Posons pour tout entier \(k\) \[S_k=\{ n\in\mathbb N\ :\quad k<\vert a_n\vert\leq k+1\}.\] Les disques fermés de centre \(a_n\) et de rayon \(1/2\) sont par hypothèse deux à deux disjoints et pour tout \(n\in S_k\) \[\overline D(a_n, 1/2)\subset \{z\in\mathbb C\ :\ k-\dfrac{1}{2}\leq \vert z\vert\leq k+\dfrac{3}{2}\}=C(0,k-1/2,k+3/2).\] (si \(k=0\), interpréter le second terme comme le disque \(D(0,3/2)\)) soit, en sommant les aires \[\text{card}(S_k)\dfrac{\pi}{4}\leq \pi\left[ \left( k+\dfrac{3}{2}\right)^2- \left( k-\dfrac{1}{2}\right)^2\right] =2\pi(2k+1)\] si \(k\in\mathbb N^\star\) et \[\text{card}(S_0)\dfrac{\pi}{4}\leq \pi \dfrac{9}{4}\] si \(k=0\). Ainsi \[\text{card}(S_0)\leq 9\quad\text{et}\quad\text{card}(S_k)\leq 8(2k+1),\ \forall\,k\in\mathbb N^\star.\] Par conséquent, pour tout \(k\in\mathbb N^\star\) \[\sum_{n\in S_k}\dfrac{1}{\vert a_n\vert^3}\leq\dfrac{\text{card}(S_k)}{k^3}\leq \dfrac{8(2k+1)}{k^3}\leq \dfrac{24}{k^2}\] car \(\overline D(a_n,1/2)\subset C(0,k-1/2,k+3/2)\) implique \(\vert a_n\vert\leq k\) et \(k\geq 1\implies 2k+1\leq 3k\). De même \(S_0\) étant fini, la somme \(\sum_{n\in S_0}\frac{1}{\vert a_n\vert^3}\) est finie et finalement \[\sum_{n\in\mathbb N}\dfrac{1}{\vert a_n\vert^3}=\sum_{k=0}^\infty\sum_{n\in S_k}\dfrac{1}{\vert a_n\vert^3}\leq \sum_{n\in S_0}\dfrac{1}{\vert a_n\vert^3}+\sum_{k=1}^\infty \dfrac{24}{k^2}<\infty\] où la sommation par paquets dans le second terme est légitime puisque la série est à termes positifs et \((S_k)_{k\geq 0}\) une partition de \(\mathbb N\).


Convergence d’une série par sommation par paquets *

9 novembre 2022 21:54 — Par Patrice Lassère

\(\alpha(n),\ (n\in\mathbb N)\) désignant le nombres de \(0\) dans l’écriture décimale de \(n\) en base \(3\). Montrer que la série entière \[\sum_{n=0}^\infty \dfrac{x^{\alpha(n)}}{n^3}\] converge si, et seulement si \(\vert x\vert<25\).



[ID: 2839] [Date de publication: 9 novembre 2022 21:54] [Catégorie(s): Suites et séries ] [ Nombre commentaires: 0] [nombre d'éditeurs: 1 ] [Editeur(s): Emmanuel Vieillard-Baron ] [nombre d'auteurs: 1 ] [Auteur(s): Patrice Lassère ]
Accordéon
Titre
Solution
Texte

Convergence d’une série par sommation par paquets
Par Patrice Lassère le 9 novembre 2022 21:54

L’entier \(n\in\mathbb N^\star\), admet exactement \(k+1\) chiffres dans son écriture en base \(3\) si, et seulement si \(3^k\leq n< 3^{k+1}\). Ainsi, si pour \(x>0\) on pose \[S_k=\sum_{n=3^k}^{3^{k+1}-1} \dfrac{x^{\alpha(n)}}{n^3}\quad\text{et}\quad T_k=\sum_{n=3^k}^{3^{k+1}-1}x^{\alpha(n)},\] alors, la série à termes positifs \(\sum_{n=0}^\infty \frac{x^{\alpha(n)}}{n^3}\) sera convergente si, et seulement si la série \(\sum_{k=0}^\infty S_k\) converge.

Maintenant, \(3^k\leq n< 3^{k+1}\) implique que \(3^{3k}\leq n^3<3^{3k+3}\) et donc \[\dfrac{T_k}{3^{3k+3}} \leq S_k< \dfrac{T_k}{3^{3k}}.\] Ainsi la convergence de la série \(\sum_{k=0}^\infty S_k\) équivaut à celle de \(\sum_{k=0}^\infty\frac{T_k}{3^{3k}}\).

Le nombre d’entiers \(n\) à \(k+1\) chiffres en base \(3\) (i.e. \(3^k\leq n< 3^{k+1}\)) tels que \(\alpha(n)=i\) est \(C_k^i2^{k+1-i}\) (il y a \(C_k^i\) possibililités pour choisir les \(i\) chiffres parmi les \(k\) (et pas \(k+1\) car le premier ne peut etre égal à \(0\)) puis sur les \(k+1-i\) restant nous avons le choix entre \(1\) et \(2\) soit encore \(2^{k+1-i}\) possibilités), par conséquent \[T_k=\sum_{i=0}^kC_k^i2^{k+1-i}x^i=2(x+2)^k\] et \[\sum_{k=0}^\infty\dfrac{T_k}{3^{3k}}=2\sum_{k=0}^\infty\left( \dfrac{x+2}{27}\right)^k\] série convergente si, et seulement si \(\vert({x+2})/{27}\vert<1\) i.e. (pour \(x>0\)) si et seulement si \(x<25\).

Remarque : Si on remplace base \(3\) par base \(k\) , le même raisonnement assure la convergence pour \(\vert x\vert < k^k-k+1\) (\(25 = 3^3-3+1\)...).


Divergence de la série \(\sum_{p\in\mathscr P}\frac{1}{p}\) *

9 novembre 2022 21:54 — Par Patrice Lassère

\(\mathscr P\) désignant l’ensemble des nombres premiers, montrer que la série \[\sum_{p\in\mathscr P}\dfrac{1}{p}\] diverge.



[ID: 2841] [Date de publication: 9 novembre 2022 21:54] [Catégorie(s): Suites et séries ] [ Nombre commentaires: 0] [nombre d'éditeurs: 1 ] [Editeur(s): Emmanuel Vieillard-Baron ] [nombre d'auteurs: 1 ] [Auteur(s): Patrice Lassère ]
Accordéon
Titre
Solution
Texte

Divergence de la série \(\sum_{p\in\mathscr P}\frac{1}{p}\)
Par Patrice Lassère le 9 novembre 2022 21:54

Notons \(p_1,p_2,\dots\) la suite croissante des nombres premiers. Si la série \(\sum_{p\in\mathscr P}\frac{1}{p}\) converge, il existe un entier \(k\) tel que \[\sum_{i\geq k+1}\dfrac{1}{p_i}<\dfrac{1}{2}\] et par suite \[\forall\,N\in\mathbb N,\qquad\sum_{i\geq k+1}\dfrac{N}{p_i}<\dfrac{N}{2}.{(\text{$\star$})}\] Nous dirons que \(p_1,p_2,\dots,p_k\) sont les petits nombres premiers, les autres \(p_{k+1},p_{k+2},\dots\) seront les grands nombres premiers. Pour \(N\in\mathbb N\), \(N_b\) sera le nombre d’entiers \(n\leq N\) admettant au moins un grand nombre premier comme diviseur et \(N_s\) lui désignera le nombre d’entiers \(n\leq N\) admettant uniquement des petits nombres premiers comme diviseur. Nous allons montrer que l’hypothèse de convergence de la série assure, pour un entier \(N\) convenablement choisi, l’inégalité \(N_b+N_s<N\) d’où la contradiction désirée puisque bien entendu \(N=N_b+N_s\).

Pour estimer \(N_b\), remarquons que \(E\left( \dfrac{N}{p_1}\right)\) est le nombre d’entiers \(n\leq N\) multiples de \(p_1\), la formule (\(\star\)) donne \[N_b\leq \sum_{i\geq k+1}E\left( \dfrac{N}{p_i}\right) <\dfrac{N}{2}.{(\text{$\star$})}\] Considérons maintenant un entier \(n\leq N\) n’admettant que des petits diviseurs premiers, et écrivons le sous la forme \(n=a_nb_n^2\)\(a_n\) est la partie non carrée. Chaque \(a_n\) se décompose donc en un produit de différents petits nombres premiers : il y a donc \(2^k\) choix possibles pour \(a_n\). Comme \(b_n\leq \sqrt n\leq \sqrt N\), il reste au plus \(\sqrt N\) choix possibles pour \(b_n\) soit \[N_s\leq 2^k\sqrt{N}.{(\bigstar)}\] On choisit enfin \(N\) suffisamment grand pour que \(2^k\sqrt{N}\leq N/2\) (i.e. \(2^{k+1}\leq \sqrt{N}\)), par exemple \(N=2^{2k+2}\) ; la formule (\(\star\)) étant vraie pour tout \(N\) elle donne avec \((\bigstar)\) : \(N_b+N_s<N\) d’où la contradiction.

Remarque : La divergence de la série \(\sum_{p\in\mathscr P}\frac{1}{p}\) est établie pour la première fois par L.Euler en 1748. La preuve ci-dessus est celle de P.Erdös (1938). Un corollaire immédiat est que l’ensemble des nombres premiers est infini.


Si \((x_j)_j\subset\mathbb R_+\) et \(\sum_j x_j=A\) alors \(\sum_j x_j^2\ \in\,]0,A^2[\) *

9 novembre 2022 21:54 — Par Patrice Lassère

Putnam, 2000.

Soient \(A\in\mathbb R_+, (x_j)_j\subset\mathbb R_+\) ; si \(\sum_j x_j=A\) quelles sont les valeurs possibles pour \(\sum_j x_j^2\) ?



[ID: 2843] [Date de publication: 9 novembre 2022 21:54] [Catégorie(s): Suites et séries ] [ Nombre commentaires: 0] [nombre d'éditeurs: 1 ] [Editeur(s): Emmanuel Vieillard-Baron ] [nombre d'auteurs: 1 ] [Auteur(s): Patrice Lassère ]
Accordéon
Titre
Solution
Texte

Si \((x_j)_j\subset\mathbb R_+\) et \(\sum_j x_j=A\) alors \(\sum_j x_j^2\ \in\,]0,A^2[\)
Par Patrice Lassère le 9 novembre 2022 21:54

Nous allons vérifier que l’ensemble des valeurs possibles pour la série \(\sum_j x_j^2\) est l’intervalle ouvert \(]0,A^2[\).

-Pour s’assurer que \(\sum_j x_j^2\subset ]0,A^2[\) on peut remarquer (la série est bien entendu convergente et à valeurs dans \(\mathbb R_+^\star\)) que pour tout \(m\in\mathbb N\) \[\left( \sum_{j=0}^m x_j\right)^2=\sum_{j=0}^m x_j^2+2\sum_{0\leq j<k\leq m} x_jx_k\] si bien qu’on peut écrire \[\sum_{j=0}^m x_j^2\leq \left( \sum_{j=0}^m x_j\right)^2 - 2\sum_{0\leq j<k\leq m} x_jx_k\leq A^2-2x_0x_1,\quad\forall\,m\in\mathbb N\] soit, si \(m\) tends vers l’infini \[\sum_{j\geq 0}x_j^2\leq A^2-2x_0x_<A^2.\]

-Il reste maintenant à s’assurer que toutes les valeurs de \(]0,A^2[\) vont être atteintes : à cet effet, on considère la suite géométrique \((x_j)_j\) de raison \(d\) variable (\(x_{j+1}/x_j=d\)). Alors \[\sum_{j\geq 0}x_j= \dfrac{x_0}{1-d}\quad\text{et}\quad\sum_{j\geq 0}x_j^2=\dfrac{x_0^2}{1-d^2}=\dfrac{1-d}{1+d}\left( \sum_{j=0}^m x_j\right)^2=\dfrac{1-d}{1+d}A^2.\] Vu cette dernière formule, lorsque \(d\) croit de \(0\) à \(1\) la quantité \((1-d)/(1+d)\) décroit de \(1\) à \(0\) et par conséquent \(\sum_j x_j^2\) va prendre toutes les valeurs entre \(0\) et \(A^2\).


Formule de Wallis et applications *

9 novembre 2022 21:54 — Par Patrice Lassère

Établir la formule de Wallis (1655) \[\dfrac{\pi}{2}=\dfrac{2.2}{1.3}\dfrac{4.4}{3.5}\dfrac{6.6}{5.7}\dots=\prod_{n\geq 1} \dfrac{4n^2}{4n^2-1}.{(\text{$\star$})}\]

En déduire que dans le jeu de pile ou face la probabilité \(p_n\) lors de \(2n\) jets successifs d’obtenir \(n\) pile et \(n\) face satifait, quand \(n\) tends vers l’infini, à l’équivalent \[p_n \underset{n\to\infty}{\sim}\dfrac{1}{\sqrt{\pi n}}.\]

En déduire la valeur de l’intégrale de Gauss \[\int_0^\infty \exp{(-t^2)}dt=\dfrac{\sqrt{\pi}}{2}.\]

En déduire la formule de Stirling (1730) \[n!\underset{n\to\infty}{\sim}\sqrt{2\pi n}\left( \dfrac{n}{e}\right)^n.\]

Du développement de Stirling \[n!=\sqrt{2\pi n}\left( \dfrac{n}{e}\right)^n\left( 1+\dfrac{1}{12n} +o\left( \dfrac{1}{n^2}\right) \right)\] préciser la formule de Stirling \[\dfrac{\pi}{2}-p_n\underset{n\to\infty}{\sim} \dfrac{\pi}{8n}\]



[ID: 2845] [Date de publication: 9 novembre 2022 21:54] [Catégorie(s): Suites et séries ] [ Nombre commentaires: 0] [nombre d'éditeurs: 1 ] [Editeur(s): Emmanuel Vieillard-Baron ] [nombre d'auteurs: 1 ] [Auteur(s): Patrice Lassère ]
Accordéon
Titre
Solution
Texte

Formule de Wallis et applications
Par Patrice Lassère le 9 novembre 2022 21:54

On reprend in-extenso des extraits du remarquable ouvrage de P. Eymard & J.P. Lafon, [EYLA].

  1. Elle est archi-classique et se trouve dans votre livre de chevet. Des intégrales de Wallis : \[I_m=\int_0^{\pi/2}\sin^m(t)dt=\begin{cases} \dfrac{1.3.5\dots (2n-1)}{2.4.6\dots(2n)}\dfrac{\pi}{2}&\quad\text{si}\quad m=2n,\\ \dfrac{2.4.6\dots(2n)}{3.5.7\dots (2n+1)}&\quad\text{si}\quad m=2n+1 \end{cases}\] on tire \[\dfrac{\pi}{2}=p_n\dfrac{I_{2n}}{I_{2n+1}}{\text{($\star$)}}\] avec \[p_n=\dfrac{2^2.4^2.6^2\dots (2n)^2}{(1.3)(3.5)\dots(2n-1)(2n+1)}=\prod_{k=1}^n\dfrac{4k^2}{4k^2-1}.\] donc, pour prouver \(\text{($\star$)}\), il reste à vérifier que \[\lim_{n\to\infty}\dfrac{I_{2n}}{I_{2n+1}}=1.\] Or, la suite \((I_n)_n\) est visiblement décroissante et par suite \[1\leq \dfrac{I_{2n}}{I_{2n+1}}\leq \dfrac{I_{2n-1}}{I_{2n+1}}=1+\dfrac{1}{2n},{\text{$\star'$}}\] ce qui achève la démonstration.

    Remarque : en combinant (\(\star\)) et \(\star'\) on a une première estimation de l’erreur \[0\leq\dfrac{\pi}{2}-p_n\leq \dfrac{\pi}{4n}.\] En fait, grâce à la formule de Stirling nous verrons que \[\dfrac{\pi}{2}-p_n\underset{n\to\infty}{\sim}\dfrac{\pi}{8n}\] en d’autres termes le produit infini \(\text{($\star$)}\) converge trés lentement.

  2. Avec les mêmes notations, nous pouvons écrire \[p_n=\dfrac{2^2.4^2.6^2\dots (2n)^2}{1^2.3^2\dots(2n-1)^2}\dfrac{1}{2n+1} =\dfrac{2^{4n}(n!)^4}{\left[ (2n)!\right]^2 }\dfrac{1}{2n+1}\underset{n\to\infty}{\sim} \dfrac{2^{4n}(n!)^4}{\left[ (2n)!\right]^2 }\dfrac{1}{2n}.\] on peut donc réecrire la formule de Wallis sous la forme \[\pi=\lim_{n\to\infty}\dfrac{2^{4n}(n!)^4}{n\left[ (2n)!\right]^2 }\] ou encore \[\dfrac{1}{\sqrt{\pi n}}\underset{n\to\infty}{\sim}\dfrac{1}{2^{2n}}\dfrac{(2n)!}{(n!)^2}.{(\bigstar)}\] Par conséquent, la probabilité d’obtenir après \(2n\) tirages d’un pièce de monnaie une nombre égal de pile et de face est \[\dfrac{\text{cas favorables}}{\text{cas possibles}}=\dfrac{C_{2n}^n}{2^{2n}}=\dfrac{1}{2^{2n}}\dfrac{(2n)!}{(n!)^2}\underset{n\to\infty}{\sim}\dfrac{1}{\sqrt{\pi n}}.\] ce qu’il nous fallait établir.

  3. On effectue dans l’intégrale de Gauss le changement de variables \(t=\sqrt{n}x\) \[G:=\int_0^\infty \exp{(-t^2)}dt=\sqrt{n}\int_0^\infty \exp{(-nx^2)}dx.\] Des inégalités (exercice !) \[0\leq 1-x^2\leq e^{-x^2},\quad 0\leq x\leq 1\quad\text{et}\quad e^{-x^2}\leq\dfrac{1}{1+x^2},\quad x\geq 0,\] on déduit \[\sqrt{n}\int_0^1(1-x^2)^ndx\leq G\leq \sqrt{n}\int_0^\infty\dfrac{dx}{(1+x^2)^n}\] et après les changements de variable \(x=\sin(u)\) dans l’intégrale de gauche et \(x=\tan(u)\) dans celle de droite on voit apparaitre nos intégrales de Wallis \[\sqrt{n}I_{2n+1}\leq G\leq \sqrt{n}I_{2n-2}\] soit \[\dfrac{2^{2n}(n!)^2}{(2n)!}\dfrac{\sqrt{n}}{2n+1}\leq G\leq\sqrt{n} \dfrac{(2n)!}{2^{2n}(n!)^2}\dfrac{2n}{2n-1}\dfrac{\pi}{2}\] et quand \(n\) tends vers l’infini, les deux extrémités de cette formule tendent vers \(\dfrac{\sqrt{\pi}}{2}\) d’après \((\bigstar)\), soit \[G:=\int_0^\infty \exp{(-t^2)}dt=\dfrac{\sqrt{\pi}}{2}.\]

  4. La preuve se décline en deux étapes : on commence par montrer qu’il existe une constante \(C>0\) telle que \[n!\underset{n\to\infty}{\sim} C\sqrt{n}\left(\dfrac{n}{e} \right)^n\] puis avec la formule de Wallis on vérifie que \(C=\sqrt{2\pi}.\)

    Pour déterminer la valeur de la constante \(C\) on reporte l’équivalent \(n!\underset{n\to\infty}{\sim} C\sqrt{n}\left(\dfrac{n}{e} \right)^n\) dans \((\bigstar)\) ce qui nous donne \[\dfrac{1}{\sqrt{\pi n}}\underset{n\to\infty}{\sim}\dfrac{(2n)!}{2^{2n}(n!)^2}\underset{n\to\infty}{\sim}\dfrac{C\sqrt{2n}\left( \dfrac{2n}{e}\right)^{2n} } {2^{2n}C^2n\left( \dfrac{n}{e}\right)^{2n}}=\dfrac{1}{C}\sqrt{\dfrac{2}{n}}\] soit \(C=\sqrt{2\pi}\), la formule de Stirling est démontrée.

  5. A suivre (page 113, exercice 23)


Série non commutativement convergente *

9 novembre 2022 21:54 — Par Patrice Lassère

Exemple d’une série convergente dont un réarrangement des termes modifie la somme. Il y a beacoup de choses à dire et d’exemples plus simples.........



[ID: 2847] [Date de publication: 9 novembre 2022 21:54] [Catégorie(s): Suites et séries ] [ Nombre commentaires: 0] [nombre d'éditeurs: 1 ] [Editeur(s): Emmanuel Vieillard-Baron ] [nombre d'auteurs: 1 ] [Auteur(s): Patrice Lassère ]
Accordéon
Titre
Solution
Texte

Série non commutativement convergente
Par Patrice Lassère le 9 novembre 2022 21:54

Il est bon de rappeler qu’il existe une constante (constante d’Euler) \(\gamma>0\) telle que \[x_n=\sum_{k=1}^n\dfrac{1}{k}=\log(n)+\gamma+\varepsilon_n\quad\text{avec}\quad\lim_{n\to\infty}\varepsilon_n=0.{(\bigstar)}\] Considérons alors \[y_n=\sum_{k=1}^n\dfrac{(-1)^{k+1}}{k}\] on a \[\begin{aligned}y_{2n}&=1-\dfrac{1}{2}+\dfrac{1}{3}-\dots+\dfrac{1}{2n-1}+\dfrac{1}{2n}\\ &= 1+\dfrac{1}{3}+\dfrac{1}{5}+\dots+\dfrac{1}{2n-1} -\dfrac{1}{2}\left(1+\dfrac{1}{2}+\dfrac{1}{3}\dots+\dfrac{1}{n} \right) \\ &= 1+\dfrac{1}{3}+\dfrac{1}{5}+\dots+\dfrac{1}{2n-1} +\dfrac{1}{2}\left(1+\dfrac{1}{2}+\dfrac{1}{3}\dots+\dfrac{1}{n} \right) -\left(1+\dfrac{1}{2}+\dfrac{1}{3}\dots+\dfrac{1}{n} \right) \\ &= 1+\dfrac{1}{2}+\dfrac{1}{3}+\dots+\dfrac{1}{2n-1}+\dfrac{1}{2n} -\left(1+\dfrac{1}{2}+\dfrac{1}{3}\dots+\dfrac{1}{n} \right) \\ &=x_{2n}-x_{n}=\log(2n)+\gamma+\varepsilon_{2n}-\log(n)-\gamma-\varepsilon_n\\ &=\log(2)+\varepsilon_{2n}-\varepsilon_n\longrightarrow\log(2)\quad\text{lorsque}\quad n\to\infty \end{aligned}\] la série \(\sum_{n\geq 1}y_n\) est donc convergente et de somme \(\log(2)\).

Groupons maintenant les termes en prenant alternativement deux termes positifs puis un terme négatif, par exemple pour les premiers termes : \[1,\ \dfrac{1}{3},\ -\dfrac{1}{2},\ \dfrac{1}{5},\ \dfrac{1}{7},\ -\dfrac{1}{4},\ \dfrac{1}{9},\ \dfrac{1}{11},\ -\dfrac{1}{6}\dots\] si on désigne par \(z_n\) la somme des \(n\) premiers termes de cette nouvelle suite \[\begin{aligned} z_{3n}&=1+\dfrac{1}{3}+\dfrac{1}{5}+\dots+\dfrac{1}{4n-1} -\left( \dfrac{1}{2}+\dfrac{1}{4}+\dots+\dfrac{1}{2n}\right)\\ &= 1+\dfrac{1}{2}+\dfrac{1}{3}+\dots+\dfrac{1}{4n}-\left( \dfrac{1}{2}+\dfrac{1}{4}+\dots+\dfrac{1}{4n}\right) -\left( \dfrac{1}{2}+\dfrac{1}{4}+\dots+\dfrac{1}{2n}\right)\\ &=x_{4n}-\dfrac{1}{2}\left( x_{2n}+x_n\right)\\ &=\log(4n)-\dfrac{1}{2}\left( \log(2n)+\log(n)\right)+\varepsilon_{4n}-\dfrac{1}{2}\left( \varepsilon_{2n}+\varepsilon_n\right)\ \longrightarrow\dfrac{3}{2}\log(2) \quad\text{lorsque}\quad n\to\infty \end{aligned}\] et la modification de l’ordre des termes a bien modifié la somme de la série.

Remarque : ...................................


Accordéon
Titre
Solution
Texte

Suites numériques, calculs d’équivalents
Par Patrice Lassère le 9 novembre 2022 21:54
  1. \((v_n)_n\) converge vers zéro et si \(\lim_{n\to\infty}v^\alpha_{n+1}-v^\alpha_n=\lambda\) on a par Cesaro :

    \[\begin{aligned} \lambda &=\lim_{n\to+\infty}{{\sum_{k=0}^{n-1}v_{k+1}^\alpha-v_k^\alpha}\over n}\\ &=\lim_{n\to\infty}\left({v_n^\alpha\over n}-{v_0^\alpha\over n}\right)\\ &=\lim_{n\to\infty}{v_n^\alpha\over n} \end{aligned}\]

    soit vu que \(\lambda\ne 0\) : \[u_n-l\underset{n\to\infty}{\sim} \left(n\lambda\right)^{1\over\alpha}.\]

  2. Passons à l’exemple : la suite \((u_n)_n\) est bien définie, décroissante minorée : elle est donc convergente vers la solution \(l\in[0,1]\) de \(\sin(l)=l,\) soit \(l=0\). Pour voir si la méthode précédente s’applique écrivons

    \[\begin{aligned} u_{n+1}^\alpha-u_n^\alpha &= \sin^\alpha (u_n)-u_n^\alpha \\ &=\left(u_n-{{u_n^3}\over 6}+o(u_n^3)\right)^\alpha-u_n^\alpha\quad(\text { car }\lim_n u_n=0)\\ &=u_n^\alpha\left(1-{{\alpha u_n^2}\over 6} +o(u_n^2)\right)-u_n^\alpha\\ &\underset{n\to\infty}{\sim}-{\alpha\over 6}u_n^{\alpha+2}\quad\text{ si }\ \alpha\ne 0. \end{aligned}\]

    Ainsi, pour \(\alpha=-2\) \[\lim_n\ u^ {-2}_{n+1}-u_n^{-2}={1\over 3}\] et par suite\(\displaystyle u_n\underset{n\to\infty}{\sim}\sqrt{3\over n}\).

    Remarque : L’exemple précédent est assez classique, pour changer vous pouvez par exemple montrer que si \((a_n)_n\subset\mathbb R_+,\ S_n:=\sum_{k=1}^n a_k^2\) et \(\lim_na_nS_n=1\) alors \(a_n\underset{n\to\infty}{\sim} n^{-1/3}\).


Irrationalité de \(e\) (2) *

9 novembre 2022 21:54 — Par Patrice Lassère

On définit la fonction exponentielle comme la solution de l’équation différentielle \(y'=y,\ y(0)=1\). Utiliser la formule de Taylor-Lagrange pour montrer que \(e:=y(1)\not\in\mathbb Q\).



[ID: 2851] [Date de publication: 9 novembre 2022 21:54] [Catégorie(s): Suites et séries ] [ Nombre commentaires: 0] [nombre d'éditeurs: 1 ] [Editeur(s): Emmanuel Vieillard-Baron ] [nombre d'auteurs: 1 ] [Auteur(s): Patrice Lassère ]
Accordéon
Titre
Solution
Texte

Irrationalité de \(e\) (2)
Par Patrice Lassère le 9 novembre 2022 21:55

On raisonne par l’absurde en suppposant \(e=p/q\in\mathbb Q\). Commencons par remarquer qu’en appliquant à la fonction exponentielle la formule de Taylor-Lagrange à l’ordre \(2\) sur \([0,1]\), il existe \(u\in]0,1[\) tel que \[e=1+1+\dfrac{1}{2}+\dfrac{e^u}{3!},\] et la fonction exponentielle étant strictement croissante \[\dfrac{5}{2}<e<\dfrac{5}{2}+\dfrac{e}{6},\] soit \(5/2<e<3\). Il en résulte que si \(e=p/q\in\mathbb Q\) nécessairement \(q>2\). Maintenant appliquons la formule de Taylor-Lagrange à l’ordre \(q\) sur \([0,1]\), il existe \(u_q\in]0,1[\) tel que \[e=1+1+\dfrac{1}{2}+\dots+\dfrac{1}{q!}+\dfrac{e^{u_q}}{(q+1)!},\] soit \[q!\left( e-\left[ 1+1+\dfrac{1}{2}+\dots+\dfrac{1}{q!}\right] \right) =\dfrac{q!e^{u_q}}{(q+1)!}\] qui implique \[0<\left\vert q!e-\sum_{k=0}^q\,q!k!^{-1}\right\vert < \dfrac{1}{q+1}<\dfrac{1}{3}<1\] ce qui est absurde puisque \(e-\sum_{k=0}^q\,k!^{-1}\in\mathbb Z\), contradiction et \(e\in\mathbb Q\).

Voir aussi page [exp1] pour une autre preuve de l’irrationalité de \(e\).


Irrationalité de \(e\) (3) *

9 novembre 2022 21:55 — Par Patrice Lassère

Soit \(\displaystyle e:=\sum_{k\geq 0}{1\over k!}=1+{1\over 1!}+{1\over 2!}\dots+{1\over n!}+r_n\). Montrer que

\[\qquad{1\over n+1}<n!r_n<{1\over n}\]

en déduire que \(\displaystyle\lim_{n\to\infty}n\sin(2\pi e n!)=2\pi\) puis que \(e\not\in\mathbb Q\).



[ID: 2853] [Date de publication: 9 novembre 2022 21:55] [Catégorie(s): Suites et séries ] [ Nombre commentaires: 0] [nombre d'éditeurs: 1 ] [Editeur(s): Emmanuel Vieillard-Baron ] [nombre d'auteurs: 1 ] [Auteur(s): Patrice Lassère ]
Accordéon
Titre
Solution
Texte

Irrationalité de \(e\) (3)
Par Patrice Lassère le 9 novembre 2022 21:55

On a \[n!r_n=n!\sum_{k\geq n+1}{1\over k!}={1\over n+1}+{1\over{(n+1)(n+2)}}+\dots\] i.e. \[n!r_n>{1\over n+1}.\] On en déduit aussi \[n!r_n< {1\over n+1}+{1\over{(n+1)^2}}+\dots={1\over n}\] soit la double inégalité. De là on tire immédiatement \[\lim_{n\to\infty}\,n\,n!r_n = 1,\ {(\bigstar)}\] puis,

\[n\sin(2\pi en!)=n\sin(2\pi n!r_n)={{\sin(2\pi n!r_n)}\over{2\pi n!r_n}}2\pi r_n\,n!n\rightarrow 2\pi {(\text{$\star$})}\]

la limite résultant de \((\bigstar)\) et (\(\star\)) via \(\sin(x)\underset{x\to 0}{\sim} x\). D’un autre côté, si \(e\in\mathbb Q\,:\ \sin(2\pi e n!)=0\) d’où le résultat.

Remarques : -On peut donner une variante (plus classique, plus rapide) basée sur les mêmes inégalités : supposons que \(e={p\over q}\) avec \(q>1\), vu ce qui précède \[0<q!\left\vert e-\sum_{k=0}^q{1\over k!}\right\vert =q!r_q<{1\over q}\] autrement dit \[q!\left( e-\sum_{k=0}^q{1\over k!}\right)\in\ ]0,1[\] ce qui est visiblement absurde car \[q!\left( e-\sum_{k=0}^q{1\over k!}\right)= q!\left( {p \over q}-\sum_{k=0}^q{1\over k!}\right)\in\mathbb N.\]

-Le même argument montre que pour toute suite \((\varepsilon_n)_n\in\mathbb \{0,1\}^{\mathbb N}\) (non identiquement nulle à partir d’un certain rang), le réel \(\sum_{n\geq 0}\dfrac{\varepsilon_n}{n!}\) est irrationel.

-Voir aussi page [exp1] pour une autre preuve de l’irrationalité de \(e\).

-Montrer que \(\pi\) est irrationnel est plus délicat, une démonstration est donnée dans l’exercice suivant.


Irrationalité de \(\pi^2\) et donc de \(\pi\) *

9 novembre 2022 21:55 — Par Patrice Lassère

Montrer que \(\pi^2\) est irrationel, en déduire que \(\pi\) est irrationel.



[ID: 2855] [Date de publication: 9 novembre 2022 21:55] [Catégorie(s): Suites et séries ] [ Nombre commentaires: 0] [nombre d'éditeurs: 1 ] [Editeur(s): Emmanuel Vieillard-Baron ] [nombre d'auteurs: 1 ] [Auteur(s): Patrice Lassère ]
Accordéon
Titre
Solution
Texte

Irrationalité de \(\pi^2\) et donc de \(\pi\)
Par Patrice Lassère le 9 novembre 2022 21:55

Considérons pour \(n\in\mathbb N^\star\) la fonction \[f_n(x)=\dfrac{x^n(1-x)^n}{n!}=\dfrac{1}{n!}\sum_{k=n}^{2n}c_kx^k,\] il n’est pas difficile de vérifier les propriétés suivantes :

\(\rightsquigarrow\quad\) \(c_k\in\mathbb N\) pour tout \(n\leq k\leq 2n\),

\(\rightsquigarrow\quad\) \(0<f_n(x)<\dfrac{1}{n!},\quad\forall\,x\in]0,1[\),

\(\rightsquigarrow\quad\) \(f_n^{(k)}(0)=0\)si \(k<n\) ou si \(k>2n\),

\(\rightsquigarrow\quad\) \(f_n^{(k)}(0)=\dfrac{k!c_k}{n!}\)si \(n\leq k\leq 2n\).

Donc \(f_n\) et toutes ses dérivées prennent des valeurs entières pour \(x=0\) ; comme de plus \(f_n\) est symétrique par rapport à \(x=1/2\), il en est de même si \(x=1\).

Supposons \(\pi^2\) rationnel égal à \(\dfrac{p}{q}\) et soit \[g_n(x)=q^n\left( \pi^{2n}f_n(x)-\pi^{2n-2}f_n''(x)+\pi^{2n-4}f_n^{(4)}(x)-\dots+(-1)^nf_n^{(2n)}(x)\right) .\] Les nombres \(g_n(0)\) et \(g_n(1)\) sont entiers et de plus \[\begin{aligned} \dfrac{d}{dx}\left( g_n'(x)\sin(\pi x)-\pi g_n(x)\cos(\pi x)\right) &=\left(g_n''(x)+\pi^2 g_n(x) \right)\sin(\pi x) \\ &=q^n\pi^{2n+2}f_n(x)\sin(\pi x)\\ &=\pi^2p^n\sin(\pi x)f_n(x). \end{aligned}\] Donc \[\pi\int_0^1p^n\sin(\pi x)f_n(x)dx= \left[\dfrac{g_n'(x)\sin(\pi x)}{\pi}-g_n(x)\cos(\pi x) \right]_0^1=g_n(0)+g_n(1)\] est un entier. Mais de l’autre coté \[0<\pi\int_0^1p^n\sin(\pi x)f_n(x)dx<\dfrac{\pi p^n}{n!}\] et \(\dfrac{\pi p^n}{n!}\) est strictement plus petit que \(1\) pour \(n\) assez grand, d’où la contradiction

Remarque : cette démonstration est due à Niven (1946), la preuve originale de l’irrationalité de \(\pi\) par Lambert date de 1766, il est chaudement recommandé de la consulter ([EYLA], page 130).


Suites, équivalents *

9 novembre 2022 21:55 — Par Patrice Lassère

Pour \(n\geq 1\) on définit l’entier \(a_n\) comme le plus petit entier tel que \[{1\over n}+{1\over n+1}+\dots +{1\over a_n}>1.\] Montrer que la suite \((a_n)_n\) est bien définie et \[\lim_{n\to\infty}{a_n\over n}=e.\]



[ID: 2857] [Date de publication: 9 novembre 2022 21:55] [Catégorie(s): Suites et séries ] [ Nombre commentaires: 0] [nombre d'éditeurs: 1 ] [Editeur(s): Emmanuel Vieillard-Baron ] [nombre d'auteurs: 1 ] [Auteur(s): Patrice Lassère ]
Accordéon
Titre
Solution
Texte

Suites, équivalents
Par Patrice Lassère le 9 novembre 2022 21:55

La divergence de la série harmonique (vers \(+\infty\)) assure l’existence de la suite \((a_n)_n\). On peut remarquer qu’une récurrence donne facilement pour tout \(n>1\)

\[{1\over n}+{1\over n+1}+\dots +{1\over 2n-1}<1 \qquad\&\qquad {1\over n}+{1\over n+1}+\dots +{1\over 3n-2}>1\]

i.e. \(2n-1<a_n<3n-2\), donc si \(\left({a_n\over n}\right)_n\) converge, sa limite sera dans l’intervalle \([2,3]\). Mais, vu la définition de \(a_n\) :

\[1<{1\over n}+{1\over n+1}+\dots+{1\over a_n}<1+{1\over a_n}\]

en comparant avec une intégrale

\[1-{1\over n}< {1\over n+1}+\dots+{1\over a_n}<\int_n^{a_n}{dt\over t}<{1\over n}+{1\over n+1}+\dots+{1\over a_n-1}\leq 1\]

i.e.

\[1-{1\over n}<\log\left({a_n\over n}\right)<1\]

le résultat suit en prenant l’exponentielle.


Divergence de la série harmonique, preuve record ? *

9 novembre 2022 21:55 — Par Patrice Lassère

Divergence de la série harmonique \(\displaystyle\sum_{n\geq 1}{1\over n}.\)



[ID: 2859] [Date de publication: 9 novembre 2022 21:55] [Catégorie(s): Suites et séries ] [ Nombre commentaires: 0] [nombre d'éditeurs: 1 ] [Editeur(s): Emmanuel Vieillard-Baron ] [nombre d'auteurs: 1 ] [Auteur(s): Patrice Lassère ]
Accordéon
Titre
Solution
Texte

Divergence de la série harmonique, preuve record ?
Par Patrice Lassère le 9 novembre 2022 21:55

Supposons quelle converge alors :

\[\begin{aligned} r:=\sum_{n\geq 1}{1\over n}=\left(1+{1\over 2}\right)+\left({1\over 3}+{1\over 4}\right)+\left({1\over 5}+{1\over 6}\right)+\dots > \left({1\over 2}+{1\over 2}\right)+\left({1\over 4}+{1\over 4}\right)+\left({1\over 6}+{1\over 6}\right) +\dots =r \ \ !! \end{aligned}\]


Divergence de la série harmonique (suite) *

9 novembre 2022 21:55 — Par Patrice Lassère

(
[rms],1993/94).

Soit \(s_n=\sum_{k=1}^n{1\over k}\) montrer que \(\displaystyle\lim_ns_{2n}-s_n=\log(2)\). Conclusion ?



[ID: 2861] [Date de publication: 9 novembre 2022 21:55] [Catégorie(s): Suites et séries ] [ Nombre commentaires: 0] [nombre d'éditeurs: 1 ] [Editeur(s): Emmanuel Vieillard-Baron ] [nombre d'auteurs: 1 ] [Auteur(s): Patrice Lassère ]
Accordéon
Titre
Solution
Texte

Divergence de la série harmonique (suite)
Par Patrice Lassère le 9 novembre 2022 21:55

On peut écrire

\[s_{2n}-s_n=\sum_{k=n+1}^{2n}{1\over k}=\sum_{k=1}^n{1\over n+k}={1\over n}\sum_{k=1}^n{1\over{1+{k\over n}}}={1\over n}\sum_{k=1}^nf\left({k\over n}\right)\ \text{où }f(t)={1\over 1+t},\]

on reconnaît là une somme de Riemann sur \([0,1]\) associée à la fonction \(f\in\mathscr C^0([0,1])\) par conséquent

\[\lim_{n\to\infty}s_{2n}-s_n=\int_0^1f(t)dt=\log(2).\]

La suite \((s_n)_n\) ne vérifie pas le critère de Cauchy, la série harmonique est donc divergente.

l’intérêt de cet exercice est bien entendu d’utiliser les sommes de Riemann, car la divergence de la série harmonique via le critère de Cauchy s’obtient classiquement (voir aussi l’exercice précédent) par

\[s_{2n}-s_n={1\over{n+1}}+\dots + {1\over 2n}\geq {1\over 2n}+\dots+{1\over{2n}}={1\over 2}.\]


Suites et sous-suites *

9 novembre 2022 21:55 — Par Patrice Lassère

\((x_n)_n\) est une suite réelle bornée. Soient \(a,b\in\mathbb R^\star_+\) vérifiant \({a\over b}\not\in\mathbb Q\) ; montrer que \[\left( (e^{iax_n})_n\ \&\ (e^{ibx_n})_n \text{ convergent }\right)\Longrightarrow\left((x_n)_n\text{ converge }\right).\]



[ID: 2863] [Date de publication: 9 novembre 2022 21:55] [Catégorie(s): Suites et séries ] [ Nombre commentaires: 0] [nombre d'éditeurs: 1 ] [Editeur(s): Emmanuel Vieillard-Baron ] [nombre d'auteurs: 1 ] [Auteur(s): Patrice Lassère ]
Accordéon
Titre
Solution
Texte

Suites et sous-suites
Par Patrice Lassère le 9 novembre 2022 21:55

Supposons \((x_n)_n\) divergente. Pour une suite bornée de réels divergente il n’y a qu’une alternative : elle possède deux sous-suites \((x_{s_n})_n, (x_{t_n})_n\) qui convergent vers deux limites distinctes \(l_1,l_2\). Vu les hypothèses

\[e^{ial_1}=\lim_ne^{iax_{s_n}}=\lim_ne^{iax_{t_n}}=e^{ial_2}\]

il existe donc \(k_1\in\mathbb N\) tel que \(ial_1=ial_2+2ik_1\pi\) i.e. \(a(l_1-l_2)=2k_1\pi\). De même avec la seconde suite il existe \(k_2\in\mathbb N\) : \(b(l_1-l_2)=2k_2\pi\). De ces deux relations on déduit aussitôt (car \(l_1-l_2\ne 0\)) que \(\displaystyle {a\over b}={{k_1}\over{k_2}}\) ce qui est bien sur absurde.


Divergence de la série \(\sum_{n\geq 1}{{\sin^2(n)}\over n}\) *

9 novembre 2022 21:55 — Par Patrice Lassère

Montrer que dans \(\{x\in\mathbb R\ :\ \sin^2(x)\leq{1\over 2}\}\) on ne trouvera jamais trois entiers consécutifs. En déduire la nature de la série \(\sum_{n\geq 1}{{\sin^2(n)}\over n}\).



[ID: 2865] [Date de publication: 9 novembre 2022 21:55] [Catégorie(s): Suites et séries ] [ Nombre commentaires: 0] [nombre d'éditeurs: 1 ] [Editeur(s): Emmanuel Vieillard-Baron ] [nombre d'auteurs: 1 ] [Auteur(s): Patrice Lassère ]
Accordéon
Titre
Solution
Texte

Divergence de la série \(\sum_{n\geq 1}{{\sin^2(n)}\over n}\)
Par Patrice Lassère le 9 novembre 2022 21:55

L’ensemble proposé est \[\bigcup_{k\in\mathbb Z}\,[-{\pi\over 4}+2k\pi,{\pi\over 4}+2k\pi]\,\cup\,[{3\pi\over 4}+2k\pi,{5\pi\over 4}+2k\pi]\] réunion disjointe d’intervalles de longueur \(<2\) donc ne contenant pas trois entiers consécutifs ; ils sont aussi séparés par des intervalles de longueur strictement plus grande que \(1\), donc contenant au moins un entier : l’assertion est donc claire.

Notre série étant à termes positifs, elle sera de même nature que celle de terme général \(v_n=u_{3n}+u_{3n+1}+u_{3n+2}\) et vu ce qui précède l’un des trois réels \(\sin^2(3n),\sin^2(3n+1),\sin^2(3n+2)\) est supérieur ou égal à \(1/2\) donc \[v_n\geq {{1}\over 2(3n+2)}\underset{n\to\infty}{\sim} {1\over 6n}\] et par comparaison notre série diverge.


Accordéon
Titre
Solution
Texte

Le critère de condensation de Cauchy
Par Patrice Lassère le 9 novembre 2022 21:55

les \(a_n\) étant positifs, il suffit de montrer que les sommes partielles se dominent mutuellement, pour cela, posons

\[s_n=a_1+a_2+\dots+a_n\quad\&\quad t_n=a_1+2a_2+\dots+2^na_{2^n}.\]

-Si \(n<2^k\) on peut écrire

\[\begin{aligned} s_n=a_1+a_2+a_3+\dots+a_n&\leq a_1+(a_2+a_3)+(a_4+a_5+\dots+a_7)\dots+(a_{2^k}+\dots +a_{2^{k+1}-1})\\ &\leq a_1+2a_2+4a_4+\dots +2^ka_{2^k}=t_k \end{aligned}\]

i.e.\[\quad {s_n<t_k,\quad \forall \,n<2^k.}\]

-Réciproquement, si \(n>2^k\) on écrit cette fois- ci

\[\begin{aligned} s_n& \geq a_1+a_2+(a_3+a_4)+\dots+(a_{2^{k-1}+1}+\dots+a_{2^k})\\ &\geq a_1+a_2+2a_4+\dots+2^{k-1}a_{2^k}\\ &\geq {a_1\over 2}+a_2+2a_4+\dots+2^{k-1}a_{2^k}={t_k\over 2} \end{aligned}\]

soit \[{2s_n\geq t_k,\quad \forall \,n>2^k}\] d’où le résultat suit.

-C’est le critère de condensation de Cauchy, il est fortement conseillé de l’appliquer aux exemples proposées : c’est immédiat et très efficace.

-Le nombre \(2\) ne joue aucun rôle particulier dans l’énonçé précédent : ce résultat est en fait un cas particulier du 1

Soient \((a_n)_n\) une suite décroissante de réels positifs, \((g_k)_k\) une suite strictement croissante d’entiers. S’il existe une constante \(M>0\) telle que \[g_{k+1}-g_k\leq M(g_k-g_{k-1}){(\text{$\star$})}\] alors les deux séries \[\sum_{k\geq 0}a_k\quad \text{et}\quad\sum_{k\geq 0}(g_{k+1}-g_k)a_{g_k}\] sont de mêmes nature.

La condition (\(\star\)) signifie que les lacunes de la suite \((g_k)\) ne croissent pas trop vite.


  1. 1  K.Knopp, Theory and Application of infinite series, Dover (1990), pages 120-121.

Suites, continuité *

9 novembre 2022 21:55 — Par Patrice Lassère

Si \(f\in\mathscr C^0([0,1],\mathbb R)\), montrer que

\[\lim_{n\to\infty} \int_0^1t^nf(t)dt=0 \quad\&\quad \lim_{n\to\infty}n\int_0^1t^nf(t)dt=f(1)\]



[ID: 2869] [Date de publication: 9 novembre 2022 21:55] [Catégorie(s): Suites et séries ] [ Nombre commentaires: 0] [nombre d'éditeurs: 1 ] [Editeur(s): Emmanuel Vieillard-Baron ] [nombre d'auteurs: 1 ] [Auteur(s): Patrice Lassère ]
Accordéon
Titre
Solution
Texte

Suites, continuité
Par Patrice Lassère le 9 novembre 2022 21:55

-\(f\) continue est bornée sur \([0,1]\) donc

\[\left\vert\int_0^1t^nf(t)dt\right\vert \leq \Vert f\Vert_\infty\int_0^1t^ndt={{\Vert f\Vert_\infty}\over n+1}\rightarrow 0.\]

-Pour la seconde limite, \(f\) étant continue en \(t=1\)

\[\forall\,\varepsilon>0,\ \exists \eta>0\,:\ 1-\eta\leq t\leq 1 \Longrightarrow \vert f(t)-f(1)\vert\leq \varepsilon,\] par conséquent

\[\begin{aligned} \left\vert n\int_0^1 t^n(f(t)-f(1))dt\right\vert &\leq \ n\left(\int_0^{1-\eta}t^n\vert f(t)-f(1)\vert dt +\int_{1-\eta}^1t^n\vert f(t)-f(1)\vert dt\right)\\ &\leq \ n\left( \,{{2\Vert f\Vert_\infty (1-\eta)^{n+1}}\over n+1}+{\varepsilon\over n+1}\right)\leq 2\varepsilon \quad\text{ pour }\quad n\geq N_\varepsilon \end{aligned}\]

i.e.

\[\lim_{n\to\infty}n\int_0^1t^nf(t)dt =\lim_{n\to\infty}n\int_0^1t^nf(1)dt =\lim_{n\to\infty} {nf(1)\over n+1}=f(1).\]


Sur le nombre d’éléments d’une suite récurrente *

9 novembre 2022 21:55 — Par Patrice Lassère

Soit \(f\ :\ [a,b]\to[a,b]\) une application continue. Pour \(x\in[a,b]\) on définit la suite \((x_n)_n\) par \(x_0=x\) et \(x_{n+1}=f(x_n)\) et on note \(T_x=\{x_n\ :\ n\in\mathbb N\}\). Montrer que \(T_x\) fermé implique \(\rm{card}(\text{T}_x)<+\infty\).



[ID: 2871] [Date de publication: 9 novembre 2022 21:55] [Catégorie(s): Suites et séries ] [ Nombre commentaires: 0] [nombre d'éditeurs: 1 ] [Editeur(s): Emmanuel Vieillard-Baron ] [nombre d'auteurs: 1 ] [Auteur(s): Patrice Lassère ]
Accordéon
Titre
Solution
Texte

Sur le nombre d’éléments d’une suite récurrente
Par Patrice Lassère le 9 novembre 2022 21:55

On procède par contraposée : \(x_n=x_m\) pour \(n>m\) assure que \(T_x\) est fini et alors bien entendu fermé ; on peut donc sans perte de généralité supposer les \(x_n\) deux à deux distincts. \(T_x=\{x_n\ :\ n\in\mathbb N\}\) fermé dans \([a,b]\) est compact : il existe une sous-suite convergente \((x_{n_k})_k\) de limite \(a\in T_x\) et par continuité de \(f\) : \(x_{n_k+1}=f(x_{n_k})\to f(a)\). Ainsi, sauf peut être pour un nombre fini, tous les éléments de \(T_x\) sont des points d’accumulation. On peut donc supposer, quitte à supprimer le nombre fini de points isolés, que tous les points sont d’accumulation. On peut.........................................................


Un exercice sur les séries numériques *

9 novembre 2022 21:55 — Par Patrice Lassère

Soient \((a_n)_n\) une suite décroissante de réels positifs telle que \(\sum_n\,a_n\) converge, \((b_n)_n\) une suite bornée de réels posititifs. Montrer que la série \[\sum_{n\geq 1}\,(b_1+\dots+b_n)(a_n-a_{n-1})\] converge.



[ID: 2873] [Date de publication: 9 novembre 2022 21:55] [Catégorie(s): Suites et séries ] [ Nombre commentaires: 0] [nombre d'éditeurs: 1 ] [Editeur(s): Emmanuel Vieillard-Baron ] [nombre d'auteurs: 1 ] [Auteur(s): Patrice Lassère ]
Accordéon
Titre
Solution
Texte

Un exercice sur les séries numériques
Par Patrice Lassère le 9 novembre 2022 21:55

Posons \(u_n=(b_1+\dots+b_n)(a_{n-1}-a_n),\ n\geq 2\) et notons \(S_N=\sum_{i=2}^N\,u_i\) ses sommes partielles. Soit \(M>0\) tel que \(\vert b_n\vert\leq M,\ \forall\,n\in\mathbb N\). La suite \((a_n)_n\) étant décroissante on a \(0\leq u_n\leq M(a_{n-1}-a_n)\leq nM(a_{n-1}-a_n)\) d’où \[0\leq S_N\leq M\sum_{i=2}^N\,n(a_{n-1}-a_n)=M\left(a_1+\sum_{i=1}^{N-1}a_i\right) \leq M\left(a_1+\sum_{i=1}^{\infty}a_i\right).\] La suite croissante \((S_N)_N\) est bornée, donc convergente, il en est donc de même de la série \(\sum_{n\geq 1}\,(b_1+\dots+b_n)(a_n-a_{n-1})=-\sum_n\,u_n=-\lim_N S_N\).


Calcul de \(\sum_{n=0}^\infty \frac{1}{F(2^n)}\) *

9 novembre 2022 21:55 — Par Patrice Lassère

[amm], 1984/7.

Montrer que \[\sum_{n=0}^\infty \dfrac{1}{F(2^n)} =\dfrac{7-\sqrt{5}}{2},\]\((F(n))_n\) est la suite de Fibonacci : \[F(0)=0, F(1)=1,\quad F(n)=F(n-1)+F(n-2),\quad n\in\mathbb N\setminus\{0,1\}.\]



[ID: 2875] [Date de publication: 9 novembre 2022 21:55] [Catégorie(s): Suites et séries ] [ Nombre commentaires: 0] [nombre d'éditeurs: 1 ] [Editeur(s): Emmanuel Vieillard-Baron ] [nombre d'auteurs: 1 ] [Auteur(s): Patrice Lassère ]
Accordéon
Titre
Solution
Texte

Calcul de \(\sum_{n=0}^\infty \frac{1}{F(2^n)}\)
Par Patrice Lassère le 9 novembre 2022 21:55

Il est bien connu (donner une référence, [monier], [deswar]...) que \[F(n)=\dfrac{a^n-b^n}{\sqrt{5}},\quad\text{avec}\ a=\dfrac{1+\sqrt{5}}{2},\ ab=-1,\] ce qui nous donne (avec l’identité remarquable \(c^2-d^2=(c+d)(c-d)\)) \[\sum_{n=0}^\infty\dfrac{1}{F(2^n)}=1+\sqrt{5}\sum_{n=1}^\infty\left[ \dfrac{1}{a^{2^n}-1}-\dfrac{1}{a^{2^{n+1}}-1}\right]=1+\dfrac{\sqrt{5}}{a^2-1}=\dfrac{7-\sqrt{5}}{2}\] aprés télescopage dans la seconde série.


Divergence de la série \(\sum_{n\geq 2}\frac{\cos(\log(\log(n)))}{\log(n)}\) *

9 novembre 2022 21:55 — Par Patrice Lassère

Putnam (1949)

En considérant les blocs \(N_l=\{\,k\in\mathbb N \ :\ 2\pi l-\pi/3\ \leq \log(\log(k))\leq 2\pi l\}\), établir la divergence de la série \(\quad\sum_{n\geq 2}\dfrac{\cos(\log(\log(n)))}{\log(n)}.\)



[ID: 2877] [Date de publication: 9 novembre 2022 21:55] [Catégorie(s): Suites et séries ] [ Nombre commentaires: 0] [nombre d'éditeurs: 1 ] [Editeur(s): Emmanuel Vieillard-Baron ] [nombre d'auteurs: 1 ] [Auteur(s): Patrice Lassère ]
Accordéon
Titre
Solution
Texte

Divergence de la série \(\sum_{n\geq 2}\frac{\cos(\log(\log(n)))}{\log(n)}\)
Par Patrice Lassère le 9 novembre 2022 21:55

L’indication est claire : si la série converge, par le critère de Cauchy elle ne peut admettre de blocs de termes consécutifs de somme arbitrairement grande. Observons donc les quantités \[S_l=\sum_{k\in N_l}\dfrac{\cos(\log(\log(n)))}{\log(n)}.\] (on peut remarquer que la croissance de \(k\mapsto \log_2(k)\) assure que \(N_k\) est bien constitué d’éléments consécutifs) On peut aussi écrire \[N_l=\{\,k\in\mathbb N : \exp(\exp(2\pi l-\pi/3))\leq k\leq \exp(\exp(2\pi l))\}\] et il en résulte \[\begin{aligned}\sharp(N_l)&\geq \exp(\exp(2\pi l))-\exp(\exp(2\pi l-\pi/3))-1\\ &=\exp(\exp(2\pi l))-\exp(\alpha\exp(2\pi l))-1\end{aligned}\]\(0<\alpha=\exp(-\pi/3)<1\). Ainsi, pour \(k\in N_l\) on a la minoration \[\dfrac{\cos(\log_2(k))}{\log(k)}\geq \dfrac{\cos(-\pi/3)}{\exp(2\pi l)}=\dfrac{1}{2\exp(2\pi l)},\] d’où \[S_l\geq \dfrac{\exp(\exp(2\pi l))-\exp(\alpha\exp(2\pi l))-1}{2\exp(2\pi l)}.\] Pour conclure il n’y a plus qu’à remarquer que \[\lim_{x\to+\infty}\dfrac{\exp(x)-\exp(\alpha x)-1}{2x}= \lim_{x\to+\infty}\dfrac{\exp(x)}{2x}\left[ 1-\exp(x(\alpha-1))-\exp(-x)\right]=+\infty\] (le terme entre crochets tendant vers \(1\) car \(0<\alpha<1\)).


Calcul d’une somme de série *

9 novembre 2022 21:55 — Par Patrice Lassère

Pour tout \(n\in\mathbb N\), \(\left[ n\right]\) désigne l’entier le plus proche de \(\sqrt{n}\). Montrer que \[\sum_{n=1}^\infty \dfrac{2^{\left[ n\right]}+2^{-\left[ n\right]}}{2^n}=3.\]



[ID: 2879] [Date de publication: 9 novembre 2022 21:55] [Catégorie(s): Suites et séries ] [ Nombre commentaires: 0] [nombre d'éditeurs: 1 ] [Editeur(s): Emmanuel Vieillard-Baron ] [nombre d'auteurs: 1 ] [Auteur(s): Patrice Lassère ]
Accordéon
Titre
Solution
Texte

Calcul d’une somme de série
Par Patrice Lassère le 9 novembre 2022 21:55

Bien remarquer que \(\left[ n\right]\) est toujours bien défini car l’équation \(\sqrt{n}=k+1/2,\ k,n\in\mathbb N\) n’a pas de solutions.

Comme souvent dans ce genre d’exercice l’astuce consiste à sommer paquets (ce qui est parfaitement légitime puisque la série est à termes positifs...sous reserve de convergence...) suivant les valeurs de \(\left[ n\right]\) : \[\begin{aligned} \left[ n\right]=k \ &\Leftrightarrow\ k-1/2<\sqrt{n}<k+1/2\\ &\Leftrightarrow\ k^2-k+1/4<n<k^2+k+1/4\\ &\Leftrightarrow\ k^2-k+1\leq n\leq k^2+k\quad (\text{car}\ n\in\mathbb N) \end{aligned}\] ce qui nous donne \[\begin{aligned} \sum_{n=1}^\infty \dfrac{2^{\left[ n\right]}+2^{-\left[ n\right]}}{2^n}&= \sum_{k=1}^\infty\left( \sum_{n=k^2-k+1}^{k^2+k}(2^k+2^{-k})2^{-n}\right) \\ &=\sum_{k=1}^\infty(2^k+2^{-k})2^{-k^2+k-1}\sum_{n=0}^{2k-1}2^{-n}\\ &=\sum_{k=1}^\infty(2^k+2^{-k})2^{-k^2+k-1}\cdot\dfrac{1-2^{-2k+2}}{1-2^{-1}}\\ &=\sum_{k=1}^\infty(2^k+2^{-k})(2^{-k^2+k}-2^{-k^2-k})\\ &=\sum_{k=1}^\infty(2^{-k(k-2)}-2^{-k(k+2)})\\ &=\sum_{k=1}^\infty 2^{-k(k-2)}-\sum_{k=1}^\infty2^{-k(k+2)}\\ &=\sum_{k=1}^\infty 2^{-k(k-2)}-\sum_{l=3}^\infty2^{-l(l-2)}\quad (l=k+2)\\ &=\sum_{k=1}^2 2^{-k(k-2)}=2+1+3. \end{aligned}\] C.Q.F.D.


À propos du produit de Cauchy *

9 novembre 2022 21:55 — Par Patrice Lassère

  1. Montrer que le produit de Cauchy des deux séries convergentes de termes général \(a_n=b_n=(-1)^{n+1}/\sqrt{n+1}\) diverge.

  2. Montrer que le produit de Cauchy des deux séries grossièrement divergentes de termes généraux \(a_0=3,\ a_n=3^n,\ b_0=-2,\ b_n=2^n,\ n\in\mathbb N^\star\) est absolument convergent.



[ID: 2881] [Date de publication: 9 novembre 2022 21:55] [Catégorie(s): Suites et séries ] [ Nombre commentaires: 0] [nombre d'éditeurs: 1 ] [Editeur(s): Emmanuel Vieillard-Baron ] [nombre d'auteurs: 1 ] [Auteur(s): Patrice Lassère ]
Accordéon
Titre
Solution
Texte

À propos du produit de Cauchy
Par Patrice Lassère le 9 novembre 2022 21:55
  1. Le terme général du produit de Cauchy des deux séries (convergentes car alternées) est \[\begin{aligned} c_n=\sum_{k=0}^n a_kb_{n-k}&=\sum_{k=0}^n\dfrac{(-1)^{k+1}}{\sqrt{k+1}}\cdot \dfrac{(-1)^{n-k+1}}{\sqrt{n-k+1}}\\ &=(-1)^n\sum_{k=0}^{n}\dfrac{1}{\sqrt{k+1}\sqrt{n-k+1}}, \end{aligned}\] donc \[\vert c_n\vert=\sum_{k=0}^{n}\dfrac{1}{\sqrt{k+1}\sqrt{n-k+1}}\geq \sum_{k=0}^n\dfrac{1}{n+1}=1\] d’où la grossière divergence du produit de Cauchy des deux séries \(\sum_n a_n\) et \(\sum_n b_n\).

  2. Nous avons cette fois-ci \(c_0=-6\) et pour \(n\in\mathbb N^\star\) \[\begin{aligned} c_n= \sum_{k=0}^n a_kb_{n-k}&=3\cdot 2^n+\sum_{k=1}^{n-1}2^k\cdot 3^{n-k}-2\cdot 3^n\\ &=3\cdot 2^n+3^n\cdot \sum_{k=1}^{n-1}\left( \dfrac{2}{3}\right)^k-2\cdot 3^n\\ &=3\cdot 2^n+3^n\cdot \dfrac{2/3-(2/3)^n}{1-2/3}-2\cdot 3^n=0, \end{aligned}\] d’où la convergence absolue du produit de Cauchy.

     Remarque :  Le produit de Cauchy deux séries absolument convergentes est absolument convergent, le produit de Cauchy d’une série absolument convergente et d’une série convergente est convergent, pour le reste tout peut arriver comme on peut le vérifier dans les exemples précédents.


\(e=\sum 1/k!\), une preuve élémentaire *

9 novembre 2022 21:55 — Par Patrice Lassère

CMJ, 1-1994.

Montrer que pour tout \(x\in]0,1]\) : \(1+ex>e^x>1+x\). En déduire que pour tout \(n\in\mathbb N\) et \(x\in]0,1]\) : \[1+x+\dfrac{x^2}{2!}+\dots+\dfrac{x^n}{n!}+\dfrac{ex^{n+1}}{(n+1)!} >e^x>1+x+\dfrac{x^2}{2!}+\dots+\dfrac{x^{n+1}}{(n+1)!}\] et enfin que \(\displaystyle\sum_{k=0}^\infty\,\dfrac{1}{k!}=e\).



[ID: 2883] [Date de publication: 9 novembre 2022 21:55] [Catégorie(s): Suites et séries ] [ Nombre commentaires: 0] [nombre d'éditeurs: 1 ] [Editeur(s): Emmanuel Vieillard-Baron ] [nombre d'auteurs: 1 ] [Auteur(s): Patrice Lassère ]
Accordéon
Titre
Solution
Texte

\(e=\sum 1/k!\), une preuve élémentaire
Par Patrice Lassère le 9 novembre 2022 21:55

Soit \(0<x\leq 1\), par le théorème des accroissement finis \[\dfrac{e^x-1}{x}=e^c,\quad c\in]0,x[,\] d’où \[1+ex>e^x>1+x,\quad\forall\, x\in]0,1].\] Intégrons maintenant ces inégalités \[\int_0^x\, (1+et)dt>\int_0^x\,e^tdt>\int_0^x\,(1+t)dt\] on a \[1+x+\dfrac{ex^2}{2}>e^x>1+x+\dfrac{x^2}{2},\quad\forall\, x\in]0,1].\] Une nouvelle intégration nous donne \[1+x+\dfrac{x^2}{2!}+\dfrac{ex^3}{3!}>e^x >1+x+\dfrac{x^2}{2!}+\dfrac{x^3}{3!},\quad\forall \,x\in]0,1],\] et ainsi de suite ; on a donc pour tout entier \(n\) : \[1+x+\dfrac{x^2}{2!}+\dots+\dfrac{x^n}{n!}+\dfrac{ex^{n+1}}{(n+1)!} >e^x>1+x+\dfrac{x^2}{2!}+\dots+\dfrac{x^{n+1}}{(n+1)!}.\] En particulier, \(x=1\) donne \[e-\dfrac{1}{(n+1)!}>1+1+\dfrac{1}{2!}+\dfrac{1}{3!}+\dots+\dfrac{1}{n!} >e-\dfrac{e}{(n+1)!},\quad\forall\,n\in\mathbb N,\] et il ne reste plus qu’à faire tendre \(n\) vers l’infini pour conclure.

 Remarque : Bien entendu on peut encore mieux exploiter l’inégalité précédente que l’on peut réecrire sous la forme : \[\dfrac{e}{(n+1)!}>e^x-\sum_{k=0}^n\dfrac{x^k}{k!}>\dfrac{1}{(n+1)!},\quad\forall\,n\in\mathbb N,\] soit \[e^x=\sum_{k=0}^\infty\dfrac{x^k}{k!},\quad\forall\, x\in]0,1].\]


Divergence douce de \(\sum_k\,1/k\log(k)\log(\log(k))\) par le TAF *

9 novembre 2022 21:55 — Par Patrice Lassère

CMJ, 5-1986.

Utiliser le théorème des accroissements finis pour établir la divergence de la série de terme général \(1/k\log(k)\log(\log(k))\). Estimer \(\sum_{m\leq k\leq n}1/k\log(k)\log(\log(k))\), conclusion ?



[ID: 2885] [Date de publication: 9 novembre 2022 21:55] [Catégorie(s): Suites et séries ] [ Nombre commentaires: 0] [nombre d'éditeurs: 1 ] [Editeur(s): Emmanuel Vieillard-Baron ] [nombre d'auteurs: 1 ] [Auteur(s): Patrice Lassère ]
Accordéon
Titre
Solution
Texte

Divergence douce de \(\sum_k\,1/k\log(k)\log(\log(k))\) par le TAF
Par Patrice Lassère le 9 novembre 2022 21:55

Le théorème des accroissement finis appliqué à \(x\mapsto \log(\log(\log(x)))\) sur l’intervalle \([k,k+1]\) assure que pour tout entier \(k\geq 3\), il existe \(c_k\in]k,k+1[\) tel que \[\dfrac{\log(\log(\log(k+1)))-\log(\log(\log(k)))}{(k+1)-k}= \dfrac{1}{c_k\log(c_k)\log(\log(c_k))},\] soit, pour tout \(k\geq 3\) \[\dfrac{1}{(k+1)\log(k+1)\log(\log(k+1))}<\log(\log(\log(k+1)))-\log(\log(\log(k)))<\dfrac{1}{k\log(k)\log(\log(k))}.\] En sommant ces inégalités pour \(3\leq m\leq k\leq n\) il vient \[\sum_{k=m+1}^{n+1}\,\dfrac{1}{k\log(k)\log(\log(k))}< \log(\log(\log(n+1)))-\log(\log(\log(m))) <\sum_{k=m}^{n}\,\dfrac{1}{k\log(k)\log(\log(k))}.\] Si \(m=3\), en faisant tendre \(n\) vers \(+\infty\) l’inégalité de droite ci-dessus assure la divergence de notre série.

Il est trés intéressant de remarquer que l’inégalité de gauche nous donne des informations sur l’extrème lenteur de la divergence de la série, par exemple la somme des \(9\) millions de termes entre \(10^6\) et \(10^7\) est majorée par (i.e. pour \(m=10^6-1,\ n=10^7-1\)) : \[\sum_{k=10^6}^{10^7}\,\dfrac{1}{k\log(k)\log(\log(k))}< \log(\log(\log(10^7)))-\log(\log(\log(10^6-1)))\approxeq 0.057.\] En procédant de même tout ceci marche en itérant autant de fois le \(\log\) que l’on veut, par exemple, on a aussi \[\sum_{k=10^6}^{10^7}\,\dfrac{1}{k\log(k)}\approxeq 0.154\ \text{et}\ \sum_{k=10^6}^{10^7}\,\dfrac{1}{k}\approxeq 2.303.\]  


;
Success message!